2021 AIME I Problems/Problem 12

Revision as of 13:39, 8 July 2021 by MRENTHUSIASM (talk | contribs) (Undo revision 157518 by MRENTHUSIASM (talk))

Problem

Let $A_1A_2A_3...A_{12}$ be a dodecagon ($12$-gon). Three frogs initially sit at $A_4,A_8,$ and $A_{12}$. At the end of each minute, simultaneously, each of the three frogs jumps to one of the two vertices adjacent to its current position, chosen randomly and independently with both choices being equally likely. All three frogs stop jumping as soon as two frogs arrive at the same vertex at the same time. The expected number of minutes until the frogs stop jumping is $\frac mn$, where $m$ and $n$ are relatively prime positive integers. Find $m+n$.

Solution

The expected number of minutes depends on the distances between the frogs, in either clockwise or counterclockwise order.

Let $E(a,b,c)$ denote the expected number of minutes for two frogs to arrive at the same vertex, such that the frogs are $a,b,$ and $c$ vertices apart, in either clockwise or counterclockwise order. We wish to find $E(3,3,3).$

Note that:

  1. $a+b+c=9$ always holds.
  2. $E(a,b,c)=0$ if and only if two frogs arrive at the same vertex.
  3. At the end of each minute, each frog has $2$ possibilities. So, there are $2^3=8$ possibilities in total.

We have the following system of equations: \begin{align*} E(3,3,3)&=1+\frac{2}{8}E(3,3,3)+\frac{6}{8}E(1,3,5), \\ E(1,3,5)&=1+\frac{4}{8}E(1,3,5)+\frac{1}{8}E(3,3,3)+\frac{1}{8}E(1,1,7), \\ E(1,1,7)&=1+\frac{2}{8}E(1,1,7)+\frac{2}{8}E(1,3,5). \end{align*} Rearranging and simplifying each equation, we get \begin{align*} E(3,3,3)&=\frac{4}{3}+E(1,3,5), &(1) \\ E(1,3,5)&=2+\frac{1}{4}E(3,3,3)+\frac{1}{4}E(1,1,7), &\hspace{12.75mm}(2) \\ E(1,1,7)&=\frac{4}{3}+\frac{1}{3}E(1,3,5). &(3) \end{align*} Substituting $(1)$ and $(3)$ into $(2),$ we obtain \[E(1,3,5)=2+\frac{1}{4}\left[\frac{4}{3}+E(1,3,5)\right]+\frac{1}{4}\left[\frac{4}{3}+\frac{1}{3}E(1,3,5)\right],\] from which $E(1,3,5)=4.$ Substituting this into $(1)$ gives $E(3,3,3)=\frac{16}{3}.$

Therefore, the answer is $16+3=\boxed{019}.$

~Ross Gao (Solution)

~MRENTHUSIASM (Reformatting and Minor Revisions)

See Also

2021 AIME I (ProblemsAnswer KeyResources)
Preceded by
Problem 11
Followed by
Problem 13
1 2 3 4 5 6 7 8 9 10 11 12 13 14 15
All AIME Problems and Solutions

The problems on this page are copyrighted by the Mathematical Association of America's American Mathematics Competitions. AMC logo.png